Select all the correct answers.
3x
If the measure of angle is 4, which statements are true?
The measure of the reference angle is 60°.
□ sin(0) = 2
The measure of the reference angle is 45°.
Otan(8) = 1
cos(8) = √2
The measure of the reference angle is 30°.

Answers

Answer 1

If the measure of angle θ is 3π/4, the true statements are:

sin(θ) = √2/2.The measure of the reference angle is 45°.

How to determine the true statements?

In Trigonometry, an angle with a magnitude of 3π/4 (radians) is equivalent to 135° (degrees) and it's found in the second quarter. Thus, we would calculate the reference angle for θ in second quarter as follows:

Reference angle = 180 - θ

Reference angle = 180 - 135

Reference angle = 45°.

Also, a terminal point for this angle θ is given by (-√2/2, √2/2) which corresponds to cosine and sine respectively. This ultimately implies that sin(θ) = √2/2.

tan(θ) = cos(θ)/sin(θ)

tan(θ) = [(-√2/2)/(√2/2)]

tan(θ) = -1

In conclusion, we can logically deduce that only options A and B are true statements.

Read more on terminal point here: brainly.com/question/4256586

#SPJ1

Complete Question:

If the measure of angle θ is 3π/4, which statements are true. Select all the correct answers.

A. sin(θ)=sqrt2/2

B. The measure of the reference angle is 45

C. The measure of the reference angle is 30

D. The measure of the reference angle is 60

E. cos(θ)=sqrt2/2

F. tan(θ)=1


Related Questions

The sum of two numbers is 79. Three times the first number added to 5 times the second number is 283. What are the two numbers?

Answers

Answer:

The two numbers are 56 and 23.

Step-by-step explanation:

Let the two numbers be A and B.

We are told that:

A+B = 79, and

2)  3A + 5B = 283

Rearrange the first expression to isolate A:

A = 79-B

Now use this value of B in the second expression:

3A + 5B = 283

3(79-B) + 5B = 283

237-3B + 5B = 283

2B = 46

B = 23

If B=23, we can find A from (1):  

A+B = 79

A+23 = 79

A = 56

CHECK:

Does A+B = 79?

56 + 23 = 79?  YES

Does 3A + 5B = 283?

  3(56) + 5(23) = 283?

 168 + 115 = 283?  YES

please help how do i this?​

Answers

The order of the numbers from the greatest to the least is

|-20| > |12.3| > |8| > |-6| > |-4| > |2 3/10|

Magnitude of numbers

From the question, we are to order the given numbers from the greatest to the least in magnitude

The given numbers are

|-20|, |-4|, |-6|, |8|, |2 3/10|, |12.3|

Since the modulo sign is included, the negative signs do not take effect.

Hence, the order of the numbers from the greatest to the least is

|-20| > |12.3| > |8| > |-6| > |-4| > |2 3/10|

Lear more on Magnitude of numbers here: https://brainly.com/question/11114137

#SPJ1

Mason and Laney run laps. The ratio of the number of laps Mason ran to the number Laney ran was 2:3.
If Laney ran 930 meters, how far did Mason run? Draw a tape diagram to determine how you fou your answer.

Answers

Using proportions, it is found that Mason ran 620 meters.

What is a proportion?

A proportion is a fraction of a total amount, and the measures are related using a rule of three.

Considering the ratio, the rule of three is given as follows:

2 laps for Mason - 3 laps for Laney.

n meters for Mason - 930 meters for Mason.

Applying cross multiplication:

3n = 930 x 2

n = 930 x 2/3

n = 620

Mason ran 620 meters.

More can be learned about proportions at https://brainly.com/question/24372153

#SPJ1

32.For the following exercises, given each set of information, find a linear equation satisfying the conditions, if possible.
32. (2, 4) and (4, 10)

Answers

Answer:

The required linear equation satisfying the given points (2,4) and (4,10) is y=3x-2

Step-by-step explanation:

Two points are given in question, (2,4) and (4,10).

It is required to find out a linear equation satisfying the points (2,4) and (4,10).

To find it out, find the slope of a line passing through these two given points. Then consider one of the points to give the linear equation of the line in the

[tex]$$\left(y-y_{2}\right)=m\left(x-x_{2}\right)$$[/tex]

Step 1 of 3

The slope of a line passing through the points (2,4) and (4,10) is given by

[tex]$$\begin{aligned}m &=\frac{y_{2}-y_{1}}{x_{2}-x_{1}} \\m &=\frac{10-4}{4-2} \\m &=\frac{6}{2} \\m &=3\end{aligned}$$[/tex]

Step 2 of 3

Now use the slope m=3 and use one of the two given points and write the equation in point-slope form:

[tex]$$\left(y-y_{2}\right)=m\left(x-x_{2}\right)$$\\ $$(y-4)=3(x-2)$$[/tex]

Distribute 3 ,

[tex]$$\begin{aligned}&y-4=3 x-3 \times 2 \\&y-4=3 x-6\end{aligned}$$[/tex]

Step 3 of 3

This linear function can be written in the slope-intercept form by adding 4 on both sides,

[tex]$$\begin{aligned}&y-4+4=3 x-6+4 \\&y=3 x-2\end{aligned}$$[/tex]

So, this is the required linear equation.

I need help pls WILL GIVE BRAINLIEST

Answers

Answer: (3,-2)

Step-by-step explanation:

[tex]-3x-7y=5\\15x+6y=33[/tex]

[tex]5(-3x-7y)=5(5)\\-15x-35y=25[/tex]

new system:

[tex]-15x-35y=25\\15x+6y=33[/tex]

add.

[tex]-29y=58\\y=-2[/tex]

plug in -2 for y

[tex]-3x-7(-2)=5\\-3x+14=5\\-3x=-9\\x=3[/tex]

system:

[tex](3,-2)[/tex]

Answer:

(3, - 2 )

Step-by-step explanation:

- 3x - 7y = 5 → (1)

15x + 6y = 33 → (2)

multiplying (1) by 5 and adding to (2) will eliminate x

- 15x - 35y = 25 → (3)

add (2) and (3) term by term to eliminate x

0 - 29y = 58

- 29y = 58 ( divide both sides by - 29 )

y = - 2

substitute y = - 2 into either of the 2 equations and solve for x

substituting into (2)

15x + 6(- 2) = 33

15x - 12 = 33 ( add 12 to both sides )

15x = 45 ( divide both sides by 15 )

x = 3

solution is (3, - 2 )

Cual es el resultado de f(x)=−7x−1

Answers

The function of the linear equation shows that the slope(m) = -7, the x-intercepts is (-1/7,0), and the y-intercepts is (0,-1)

What is the function of f(x) of a linear equation?

The function of a linear equation takes the form y = ax + b. In this situation, the values of y can be determined when x = 0, and the values of x can be determined when y = 0

From the given information:

y = f(x) = -7x - 1

We can determine the:

Slope (m)x-intercepts, andy-intercepts.

y = -7x - 1

Slope (m) = -7

Set the values of y = 0 to determine the x-intercepts.

0 = -7x - 1

7x = - 1

x = -1/7

x-intercepts = (-1/7, 0)

Set the values of x = 0 to find the y-intercepts.

y = -7(0) - 1

y = - 1

y-intercepts = (-1, 0)

Learn more about the function of a linear equation here:

https://brainly.com/question/15602982

#SPJ1

What is the L.C.M of: 6(a+b)+3

Answers

Answer:

I hope this is the right answer

Write an expression for cotA

Answers

Answer:

[tex]cotA=\frac{2x}{5}[/tex]

Step-by-step explanation:

Answer:

[tex]\text{cotA} =\frac{2x}{5}[/tex]

Step-by-step explanation:

cotA = (Adjacent side) ÷ (Opposite side)

The measure of the Adjacent side = 2x

The measure of the Opposite side = 5

Then

[tex]\text{cotA} =\frac{2x}{5}[/tex]

very hard problem someone help

Answers

Answer:

Everything in the box currently is correct but the bottom two numbers are just -3 and -4

Step-by-step explanation:

What is the approximate total length of iron edging needed to create the square frame and the two diagonals? 43.5 inches 50.9 inches 54.0 inches 61.5 inches

Answers

Total length of iron edging needed to create the square frame is 43.5 inches

Important properties of square:

All four interior angles are equal to 90°

All four sides of the square are congruent or equal to each other.

The opposite sides of the square are parallel to each other.

The diagonals of the square bisect each other at 90°

The two diagonals of the square are equal to each other.

Diagonal of square = [tex]\sqrt{2}[/tex]a

9 = [tex]\sqrt{2}[/tex]a

a = [tex]\frac{9}{\sqrt{2}}[/tex]

a = 6.36 inches

Iron edging needed to create the square frame = 4*6.36 = 25.44 inches

iron edging needed to create two diagonals = 2*9 = 18 inches

total length = 18 + 25.44

                   = 43.44 ~ 43.5 inches

Thus, Total length required is 43.5 inches

To learn more about Lengths visit:

https://brainly.com/question/1476578

#SPJ4

Use the properties of 30-60-90 and 45-45-90 triangles to solve for x in each of the problems below. Then decode the secret message by matching the answer with the corresponding letter/symbol from the exercises.

Answers

The trigonometric function gives the ratio of different sides of a right-angle triangle. The given problems can be solved as given below.

What are Trigonometric functions?

The trigonometric function gives the ratio of different sides of a right-angle triangle.

[tex]\rm Sin \theta=\dfrac{Perpendicular}{Hypotenuse}\\\\\\Cos \theta=\dfrac{Base}{Hypotenuse}\\\\\\Tan \theta=\dfrac{Perpendicular}{Base}\\\\\\Cosec \theta=\dfrac{Hypotenuse}{Perpendicular}\\\\\\Sec \theta=\dfrac{Hypotenuse}{Base}\\\\\\Cot \theta=\dfrac{Base}{Perpendicular}\\\\\\[/tex]

where perpendicular is the side of the triangle which is opposite to the angle, and the hypotenuse is the longest side of the triangle which is opposite to the 90° angle.

1st.) x = 5 /Sin(30°)

x = 10

!) sin(45°) = 4/x

x = 4/sin(45°)

x = 4√2

I) Cos(45°) = √3 / x

x = √3 / Cos(45°)

x = √6

E) Tan(60°) = 3√3 / x

x = 3√3 / 3

W) For isosceles right-triangle, the angle made by the legs and the hypotenuse is always 45°.

x = 45°

N) x² + x² = (7√2)²

x = 7

V) Tan(60°) = 7 / x

x = 7√3/3

K) x² + x² = (9)²

x = 9/√2

Y) Sin(60°) = 7√3/x

x = 14

M) Sin(30°) = x/11

x = 11/2

T) Sin(45°) = x/√10

x = √5

A) x + 2x + 90° = 180°

x = 30°

O) Sin(45°) = √2 / x

x = 2

R) Tan(30°) = x / 4

x = 4/√3 = 4√3 / 3

S) Sin(60°) = x / (10/3)

x = 5√3 / 3

Learn more about Trigonometric functions:

https://brainly.com/question/6904750

#SPJ1

Question is in the picture, please help me out!!

Answers

The arithmetic sequences are as follow:

What is Arithmetic Sequence?

An arithmetic sequence in algebra is a sequence of numbers where the difference between every two consecutive terms is the same.

1) t(n) = 5n + 4

t(1) = 9, t(2) = 14, t(3) = 19

So,

9,14,19,...

d= 14-9 = 5

d= 19-14 =5

Hence, it is an AP

2) 1, 2, 4, 8 , 16

Hence, it is not an AP

3) 3, 6, 9 ,...

It is an AP

4)It is given that it is an AP

5) tn =  2*3^n

 t1= 6, t2= 18, t3= 54

So, 6, 18, 54,...

Hence, it is not an AP

6) 3 , 1, 1/3,...

It is not an AP

7) t(n+1)= 6*t(n)

t(1) = -1

t(2)=-6

t(3)= -36

Hence, it is not an AP

8) -3, 1, 5, 9

Hence, it is an AP.

9) 1, 4, 9,...

Hence, it not an AP

10) 2,1,0,1,2,...

It is not an AP

11) t(n)= -2n-5

t(1)= -7, t(2)= -9, t(3)= -11

Hence, it is an AP

12) tn= (1/2)^n

 t1= 1/2, t2= 1/4, t3= 1/8

It is not an AP

Learn more about AP here:

https://brainly.com/question/24873057

#SPJ1

is 7.666 a rational number

Answers

Answer:

Yes, it's a rational number

Step-by-step explanation:

Simplify
50(x^2+7x+6)

Answers

Distribute [tex]50[/tex] to every term inside the parentheses.

[tex]50\cdotx^2+50\cdot7x+50\cdot6[/tex]

Simplify with multiplication.

[tex]\huge\boxed{50^2+350x+300}[/tex]

You can't simplify further by adding, so leave the answer as-is.

Answer:

50x^2 + 350x + 300

Step-by-step explanation:

Original Equation:

50(x^2+7x+6)

Distribute 50 to all 3 #'s:

50x^2 + 350x + 300

What is the answer to the question down below

Answers

The equation which could be used to find the value of x is; (x+10) +(x-6) = 10.

Which equation can best be used to find the value of x?

According to the task content, it follows that the lines WX and WY both amount to the line XY which joins points X and Y.

Hence, it follows that the equation which can be used to find the value of x is; (x+10) +(x-6) = 10.

Read more on equations;

https://brainly.com/question/2972832

#SPJ1

what is the value of x and y

Answers

Answer:

x = 15° , y = 125°

I hope this helps u

(3x+10)+(8x+5)=180 ( corresponding angle, TR//PA )
11x+15=180
x=(180-15)/11
= 15
(3x+10)+5y=180 ( corresponding angle, TP//RA )
sub x=15
3(15)+10+5y=180
55+5y=180
y=(180-55)/5
=25

Therefore, x=15 y=25

Which statement is not true about the data shown by the box plot below?

a.The range is 67.
b. The data point 5 lies outside the range of the data.
c. Three fourths of the data is less than 68.
d. Half the data lies between 51 and 68.

Answers

based on the options given, one thing that isn't true of the box plot is that d. Half the data lies between 51 and 68.

what is false about the box plot?

the box plot has a range of 67:

= 90 - 23

= 67

the data point 5 lies outside the data set.

3/4 of the data is less than 68 which is the 3rd quartile.

this means that the false statement is that half of the data is between 57 and 68.

find out more on box plots at https://brainly.com/question/14277132.

#SPJ1

Help please I'll mark you as brainlist if the answer is correct

if (x + q) is a factor of two polynomials x2 + px + q and x2 + mx + n, then prove that a = n-q m-p​

Answers

Answer:

factor polynomial is x+q so x=-q

putting the value of x in both equations

eqn 1)a^2+p(-a)+q=0

a^2 -ap+q=0

eqn 2)a^2+m(-a)+n=0

a^2-am+n=0

Since both are equal to zero,we can write:

a^2-ap+q=a^2-am+n

-ap+am=n-q

a(-p+m)=n-q

a=(n-q)/(m-p)

Explanation:

Since (x + a) is a factor of the two polynomials, when x = -a, the polynomials should both equal 0 (this is known as the factor theorem).

First equation:

[tex]x^{2} + px + q[/tex]

When x = -a:

[tex](-a)^{2} + p(-a) + q = 0\\\\a^{2} -pa + q = 0[/tex]

Second equation:

[tex]x^{2} + mx + n[/tex]

When x = -a:

[tex](-a)^{2} + m(-a) + n = 0\\\\a^{2} -ma + n = 0[/tex]

• As both equations equal 0, we can equate them:

[tex]a^{2} -pa + q = a^{2} -ma + n \\\\-pa + ma = n - q\\\\a(m - p) =n -q\\\\a = (n - q)/ (m - n)[/tex][proven]

How is the relationship between the length and area of the rectangle different from other kinds of relationships we've seen before ?

Answers

The area of a rectangle = Length × Breadth

The area of a rectangle is the product of the breadth and length of a rectangle. A = L * W, where A is the area, L is the length, W is the width or breadth.We must determine the product of a rectangle's length and breadth in order to get its area.Area of a rectangle is equal to length times width.For instance, if a rectangle has a length of 50 cm and a width of 7 cm, its area is equal to 350 square cm (50 × 7 square cm).Every angle of a rectangle measures 90°. Opposite sides are equal and parallel. It has 2 diagonals of equal length.

Learn more about Rectangle

brainly.com/question/14546734

#SPJ4

A rectangular tank has its width exactly half its length. That the height of the tank is 3M and the length is 8m. What is the volume of the tank in M3?

Answers

Answer:

Volume = 96m^3

Step-by-step explanation:

If the length of the tank is 8 meters, then the width must be 4 meters, because the question said the width was half the length. Since the height, 3 meters, was also given, we can find the volume.

Vol = L × w × h

We multiply together the length, width and height.

Vol = 8 × 4 × 3

= 96

The volume is 96 meters^3

Please help quickly if you can ! :)

Answers

Answer:

y=49x

Step-by-step explanation:

As you can see from the graph, the distance from home increases by 49 miles per hour driven.

Therefore the distance driven for any given time in hours is 49x

Therefore y = 49x

Taseem’s loose change consists of dimes and quarters. When you ask her how many of each type of coin she has, she gives you the following clues: 0.1x+0.25y=19.9 and x+y=100 What can you conclude? Select all that apply.

Answers

The statement first, and the statement second are correct because the number of dimes is 34, and the number of quarters is 66

What is a linear equation?

It is defined as the relation between two variables, if we plot the graph of the linear equation we will get a straight line.

If in the linear equation, one variable is present, then the equation is known as the linear equation in one variable.

We have:

0.1x + 0.25y = 19.9 and

x + y = 100

Let x = number of dimes

y = number of quarters

After solving the above system of equations by substitution method.

[tex]\rm \dfrac{398-5y}{2}+y=100[/tex]

y = 66

[tex]\rm x=\dfrac{398-5\cdot \:66}{2}[/tex]

x = 34

Thus, the statement first, and the statement second are correct because the number of dimes is 34, and the number of quarters is 66

Learn more about the linear equation here:

brainly.com/question/11897796

#SPJ1

which coordinate pair is a solution of the linear equation below -3x+4y=19

Answers

The coordinate pair that is a solution of the linear equation -3x+4y=19 is (0, 19)

How to determine the ordered pair?

The linear equation is given as:

-3x + 4y= 19

Set x = 0

So, we have:

-3(0) + 4y = 19

Evaluate

4y = 19

Divide by 4

y = 19/4

Hence, the coordinate pair that is a solution of the linear equation -3x+4y=19 is (0, 19)

Read more about linear equations at:

https://brainly.com/question/14323743

#SPJ1

please help please and thank you

Answers

Answer:

B or the second option

Step-by-step explanation:

We can label the diagram parts A-E to fully represent the situation in the question.

A: Julie's ascent up the mountain (moving/increasing)

B: Julie drinking hot chocolate (stationary/constant)

C: Julie's descent down the mountain (moving/decreasing)

D:  Julie rests (stationary/constant)

E:  Julie's final descent down the mountain (moving/decreasing)

Write a proof
given: angle 1 and angle 3 are supplementary
prove: m is parallel to n

Answers

ABCDEDJGHIKLMNOP

sorry just wanted to see if the bold thing works

A cheerleading team plans to sell t-shirts as a fundraiser. The team's
goal is to make a profit of at least $1248. The profit on each t-shirt sold
is $6.50. The team's goal is written as the inequality shown below.
6.50t ≥1248 where t=number of t-shirts sold
Which graph represents the solution to this inequality?

Answers

Answer:

You didn't include any choices.....see below

Step-by-step explanation:

Bryan is asked to write an inequality to represent the graph.

A number line going from negative 16 to negative 10. An open circle is at negative 14. Everything to the right of the circle is shaded.
He writes Negative 14 less-than-or-equal-to x.

Which error did Bryan make?
Bryan must write the inequality with the variable on the left side of the relation symbol.
Bryan chose a relation symbol that does not point in the same direction as the ray.
Bryan included –14 in his solution set.
Bryan used the wrong number in his inequality.

Answers

The error that Bryan has done in this inequality is that he included –14 in his solution set.

How to point out the mistake made

What Bryan has to do would be to write this inequality in such a way that the variable would appear on the left.

But the symbol that he has chosen does not move in this same direction as the graph.

The -14 should not be inclusive in the graph because it is not a part of the graph solution.

He used the wrong number in this inequality.

Read more on inequalities here:

https://brainly.com/question/24372553

#SPJ1

Answer:

its c

Step-by-step explanation:

on edge

What is the asymptote of the function g(x) = 5•2^3x + 4 shown on the graph?

A: y=4
B: y=5
C: y=0
D: y = 3

Answers

y=4

Hope this helps :)

in triangle abc and triangle def angle a=angle d and angle b= angle e and ab=ef. will the 2 triangles be congruent ? justify your answer

Answers

Answer:

Step-by-step explanation:

no, you don't have the measure of at least one side, so having the same angles is similar but not congruent

On a coordinate plane, parallelogram A B C D, parallelogram A prime B prime C prime D prime, and parallelogram A double prime, B double prime, C double prime, D double prime are on the graph.
In the pre-image of the parallelogram, the slope of AD is −3 and the slope of BC is −3. The image is translated down 7 units and then reflected over the y−axis.

The slope of A"D" is
.

The slope of B"C" is
.

What can you determine about the line segments A"D" and B"C"?

Answers

Using translation concepts, we have that:

The slope of A"D" is of 3.The slope of B"C" is 3.The lines are parallel.

What is a translation?

A translation is represented by a change in the function graph, according to operations such as multiplication or sum/subtraction in it's definition.

When the function is translated down, the slope remains constant, while when the function is reflected over the y-axis, the slope is multiplied by -1, hence both lines will have a slope of 3, and since they have the same slope, they are parallel.

More can be learned about translation concepts at https://brainly.com/question/4521517

#SPJ1

Other Questions
Chase Company has 10 employees, who earn a total of $2,200 in salaries each working day. They are paid on Monday for the five-day work week ending on the previous Friday. Assume that year ended on December 31, which is a Wednesday, and all employees will be paid salaries for five full days on the following Monday. The adjusting entry needed on December 31 is: Find the demand function for the marginal revenue function. Recall that if no items are sold, the revenue is 0.R'(x) = 599 - 0.21sqrt(x)What is the integral that is needed to solve the problem and the demand function? How did Building U.S. defenses help lead the U.S Into World War 2 A 4y/o with a rare genetic disorder demonstrates bilateral excessive pronation due to ligamentous laxity and hypotonia. She is able to clear B feet during swing phase without issue. What is the most appropriate orthotic for this child A dog starts chasing gretchen. when her brain sends a signal to her muscles to run, which cells are at work? sensory motor interneuron leukocyte I NEED HELP ASAPCiting examples from the novel, describe what Annes life was like before entering the Secret Annex.The book tittle is "Anne Frank The Diary of a Young Girl " Write the first five terms of the sequence. an = n 2 + 2 2, 3, 6, 11, 18 3, 6, 9, 12, 15 3, 6, 11, 18, 27 If you answer this question i will give you 100 points!!!!!!!!Write a short dialogueboth sides of the conversationin Spanish between you and your classroom friend Celeste in complete sentences. Use the vocabulary from this lesson and the following suggestions as a guide for your answer: You may copy and paste the accented and special characters from this list if needed: , , , , , , , , , , , , , , *Note: The sample sentences/questions in parentheses are just a guide to help you form your sentences/questions. You must come up with your own original answers keeping academic integrity intact.Greet your friend and ask how she is doing in one sentence. (e.g., Hi, how are you?)Have your friend respond how she is doing and ask you how you are doing. (e.g., I am well, thanks, and you?)Respond to your friend that you are well, in one sentence. (e.g., I am well, thanks.)Give a farewell. (e.g., See you later.) Determine if the given ordered triple is a solution of the system.(4, -5, -4)4x + 2y + z = 25x - 4y - z = 443x + y + 4z = -9 The diameter of a sphere isa chord that passes through the center of the sphere.a fixed point equidistant from all points on the surface of the sphere.a line segment from the center point to a point on the sphere.a three-dimensional circle in which all points are equidistant from a fixed point. 7Solve the system of inequalities by graphing.2x + 5y < 25y < 3x-25x - 7y < 14 DUE NOW Write the questions. Follow the example given below. Ex. Did you wake up early? No I didnt. I woke up late. 1. __________________________________________________ at 7:00? No, we didnt. We met at 8:00. 2. ____________________________________________ at the pool? No, he didnt. He swam in the lake. 3. _______________________________________________ apples? No, she didnt. She bought oranges. 4. ______________________________________________ French? No, they didnt. They spoke Spanish. 5. _______________________________________________ to school? No, I didnt. I drove to the office. 6. _____________________________________________________ on time? No, he didnt. He came late. 7. _____________________________________________ to Paris? No, she didnt. She flew to Rome. 8. __________________________________________________ your purse? No, I didnt. I lost my wallet. 9. ____________________________________________ a big fish? No, he didnt. He caught a small one. 10. ___________________________________________________ a cake? No, she didnt. She made a pie. 11. ______________________________________________________ 8 hours? No, I didnt. I slept 9 hours. 12. ________________________________________________ their mother? No, they didnt. They told me. 13. _______________________________________________ on the sofa? No, we didnt. We sat on chairs. 14. __________________________________________________ a skirt? No, she didnt. She wore a dress. 15. _________________________________________________ the windows? No, I didnt. I shut the door. 16. ______________________________________ at the front of the room? No, she didnt. She stood at the back of the room. Find m/HFG if m/EFG = 2x + 52, m/EFH = 21, andm/HFG = x + 31. Hi please help, tysm. Answer is in nmWhat is the wavelength of a photon iftheenergy is 7.41 x 10-19 J? (h= 6.626 x 10-34 J.s) hi who know how to do this 3 question?,I will mark brainlest for good answer. He decides to write about an event involving odysseus. which are effective research questions to best develop his topic? Need answer & explanation! Please & thank you! How many moles are in 213 mg of calcium fluoride, CaF2?2.73 mols0.00273 mols16.6 mols1.66 mols help me pls and if u can explain too. Some individuals with severe allergic reaction use what medication in an emergency?.